How to Prove an Upper Bound for a Set of Real Numbers?

Click For Summary
The discussion focuses on proving that if b is the supremum of a set A of real numbers, then for any c less than b, there exists an element a in A such that a is greater than c. Two cases are considered: when b is attained by A and when it is not. In the latter case, a proof by contradiction is suggested, indicating that if c is greater than or equal to all elements a in A, it contradicts the definition of b as the least upper bound. The conversation also highlights a potential misunderstanding regarding the conditions under which b can be considered the supremum, emphasizing the need for clarity in the statement of the problem. Overall, the thread seeks to clarify the conditions for establishing upper bounds in real number sets.
matematiker
Messages
3
Reaction score
0

Homework Statement


Let A be a set of real numbers. If b is the supremum (least upper bound) of the set A then whenever c<b there exist an a in A such that a>c.


Homework Equations





The Attempt at a Solution



I considered two cases. The first one when the supremum b is attained by the set A. In this case there exists an a belonging to A such that a=b and the statement is proved.

In the second case the supremum is not attained by the set A, so for all a that belong to A, a<b. Here is where I get stucked. I cannot come up with an idea of an a larger than c but smaller than b.

Any hint in the right direction will be very much appreciated. Thank you !
 
Physics news on Phys.org
How about a proof by contradiction? If for c≥a for all points a in A, what does that tell you about the least upper bound?
 
clamtrox said:
How about a proof by contradiction? If for c≥a for all points a in A, what does that tell you about the least upper bound?
That would mean b is not the least upper bound since c is smaller than b and greater or equal to any a in A which is a contradiction since by hypothesis b is the supremum of A.

Awesome !
Thank you clamtrox !
 
matematiker said:
That would mean b is not the least upper bound since c is smaller than b and greater or equal to any a in A which is a contradiction since by hypothesis b is the supremum of A.

Awesome !
Thank you clamtrox !



Now I am trying to prove the statement in the other direction:
Let a,b,c be reals and let A be a set of real numbers. If c<b and there is an a in A such that a>c then b is the supremum of A.

From the givens I know that:
c<b
There is an a in A such that a>c.

What I need to prove is that for any a in A, a<=b.
I took a>c from the givens but that is where I get stucked because I do not know how to show that this a is greater or equal to b.

Do you have any hint?
Thank you in advance !
 
matematiker said:
Let a,b,c be reals and let A be a set of real numbers. If c<b and there is an a in A such that a>c then b is the supremum of A.

There's something wrong here... Let a=1, A={1}, c=0 and b=42. Then c<b and a>c exists, yet b is not the supremum. Maybe it should say "If for all c<b there exists a in A s.t. a>c ... "
 
Question: A clock's minute hand has length 4 and its hour hand has length 3. What is the distance between the tips at the moment when it is increasing most rapidly?(Putnam Exam Question) Answer: Making assumption that both the hands moves at constant angular velocities, the answer is ## \sqrt{7} .## But don't you think this assumption is somewhat doubtful and wrong?

Similar threads

  • · Replies 1 ·
Replies
1
Views
1K
  • · Replies 1 ·
Replies
1
Views
2K
  • · Replies 11 ·
Replies
11
Views
3K
  • · Replies 6 ·
Replies
6
Views
2K
  • · Replies 3 ·
Replies
3
Views
2K
  • · Replies 4 ·
Replies
4
Views
2K
  • · Replies 8 ·
Replies
8
Views
3K
  • · Replies 4 ·
Replies
4
Views
1K
  • · Replies 7 ·
Replies
7
Views
3K
  • · Replies 33 ·
2
Replies
33
Views
2K